Όριο ολοκληρωμάτων

Ασκήσεις μαθηματικών προπτυχιακού επιπέδου στις οποίες πρέπει, επιπλέον, να υπάρχει καταληκτική ημερομηνία. Μέχρι αυτήν την ημερομηνία οι απαντήσεις δίνονται ΜΟΝΟ από φοιτητές. Μετά το πέρας αυτής, μπορούν να απαντήσουν όλα τα μέλη.

Συντονιστής: Demetres

Κανόνες Δ. Συζήτησης
Ασκήσεις μαθηματικών προπτυχιακού επιπέδου στις οποίες πρέπει, επιπλέον, να υπάρχει καταληκτική ημερομηνία. Μέχρι αυτήν την ημερομηνία οι απαντήσεις δίνονται ΜΟΝΟ από φοιτητές. Μετά το πέρας αυτής, μπορούν να απαντήσουν όλα τα μέλη.
Άβαταρ μέλους
M.S.Vovos
Δημοσιεύσεις: 931
Εγγραφή: Παρ Φεβ 27, 2015 5:45 pm

Όριο ολοκληρωμάτων

#1

Μη αναγνωσμένη δημοσίευση από M.S.Vovos » Παρ Φεβ 15, 2019 6:51 pm

Έστω η συνεχής συνάρτηση f:\left [ -1,1 \right ]\longrightarrow \mathbb{R}. Να αποδείξετε ότι:

\displaystyle{\lim_{n\rightarrow \infty }\frac{\displaystyle \int_{-1}^{1}f(x)\left ( 1-x^{2} \right )^{n}\textup{d}x}{\displaystyle \int_{-1}^{1}\left ( 1-x^{2} \right )^{n}\textup{d}x}=f(0)} Φιλικά,
Μάριος



Μέχρι 17/02/2019.


Είναι αυταπάτη ότι η νεότητα είναι ευτυχισμένη, μια αυταπάτη αυτών που την έχουν χάσει. W. Somerset Maugham

Λέξεις Κλειδιά:
sot arm
Δημοσιεύσεις: 222
Εγγραφή: Τρί Μάιος 03, 2016 5:25 pm

Re: Όριο ολοκληρωμάτων

#2

Μη αναγνωσμένη δημοσίευση από sot arm » Κυρ Φεβ 17, 2019 2:55 pm

M.S.Vovos έγραψε:
Παρ Φεβ 15, 2019 6:51 pm
Έστω η συνεχής συνάρτηση f:\left [ -1,1 \right ]\longrightarrow \mathbb{R}. Να αποδείξετε ότι:

\displaystyle{\lim_{n\rightarrow \infty }\frac{\displaystyle \int_{-1}^{1}f(x)\left ( 1-x^{2} \right )^{n}\textup{d}x}{\displaystyle \int_{-1}^{1}\left ( 1-x^{2} \right )^{n}\textup{d}x}=f(0)} Φιλικά,
Μάριος



Μέχρι 17/02/2019.
Βάζω μια λύση παρακάτω με κάποια επιφύλαξη για τυχόν λογιστικό σφάλμα, έστω:

\displaystyle{L(f(x))=\lim_{n\rightarrow \infty }\frac{\displaystyle \int_{-1}^{1}f(x)\left ( 1-x^{2} \right )^{n}\textup{d}x}{\displaystyle \int_{-1}^{1}\left ( 1-x^{2} \right )^{n}\textup{d}x}}

Παρατηρώ ότι είναι γραμμική, πράγματι:

\displaystyle{L(cf+g)=cL(f)+L(g), c \in \mathbb{R}} , προκύπτει απλά με πράξεις, και:

\displaystyle{L(c)=\lim_{n\rightarrow \infty }\frac{\displaystyle \int_{-1}^{1}c\left ( 1-x^{2} \right )^{n}\textup{d}x}{\displaystyle \int_{-1}^{1}\left ( 1-x^{2} \right )^{n}\textup{d}x}=c \lim_{n\rightarrow \infty }\frac{\displaystyle \int_{-1}^{1}\left ( 1-x^{2} \right )^{n}\textup{d}x}{\displaystyle \int_{-1}^{1}\left ( 1-x^{2} \right )^{n}\textup{d}x}=c}

Εξετάζουμε τώρα μονώνυμα της μορφής x^{l} , αν l περιττός άμεσα το όριο μηδέν αφού η συνάρτηση εντός του ολοκληρώματος του αριθμητή είναι περιττή,και το ολοκλήρωμα είναι σε συμμετρικό γύρω από το 0 διάστημα.

Αν πάλι l άρτιος προκύπτει με παραγοντική:

\displaystyle{\int_{-1}^{1}x^{2k}(1-x^{2})^{n}dx=\frac{2n}{2k+1}\int_{-1}^{1}x^{2k+2}(1-x^{2})^{n}dx=...=\frac{2^{n+1}n!}{\prod_{i=0}^{n}(2k+2i+1)}}

Ο παρονομαστής είναι εφαρμογή του παραπάνω για κ=0, έχω λοιπόν για το κλάσμα:

\displaystyle{L(x^{2k})=\lim_{n\rightarrow +\infty}\frac{\prod_{i=0}^{n}(2i+1)}{\prod_{i=0}^{n}(2k+2i+1)}=\lim_{n\rightarrow +\infty}\frac{\prod_{i=1}^{n}(2i+1)}{\prod_{i=0}^{n}(2k+2i+1)}= 0}

Από τα παραπάνω για κάθε πολυώνυμο έχω:
\displaystyle{L(p(x))=p(0)}
αφού όλες οι δυνάμεις του x δίνουν μηδέν και ο σταθερός όρος ισούται με p(0)

Έστω ε>0 τυχόν, η f είναι συνεχής στο [-1,1] άρα για το δοθέν ε υπάρχει πολυώνυμο έτσι ώστε:

\displaystyle{|f(x)-p(x)|<\frac{\epsilon}{2}} (Weierstrass)

Άρα:
\displaystyle{|L(f(x)-f(0))|=|L(f(x)-f(0))-L(p(x)-p(0))|=|L(f(x)-p(x))+L(f(0)-p(0)))|\leq}

\displaystyle{|L(f(x)-p(x))|+|L(f(0)-p(0))|\leq L(|f(x)-p(x)|)+L(|f(0)-P(0)|) <\frac{\epsilon}{2}+\frac{\epsilon}{2}=\epsilon}}

και τελειώσαμε, ξαναλέω με κάποια επιφύλαξη για λογιστικό σφάλμα, αλλά νομίζω η ιδέα είναι η σωστή.


Αρμενιάκος Σωτήρης
ΠΑΠΑΔΟΠΟΥΛΟΣ ΣΤΑΥΡΟΣ
Δημοσιεύσεις: 3600
Εγγραφή: Πέμ Φεβ 27, 2014 9:05 am
Τοποθεσία: ΧΑΛΚΙΔΑ- ΑΘΗΝΑ-ΚΡΗΤΗ

Re: Όριο ολοκληρωμάτων

#3

Μη αναγνωσμένη δημοσίευση από ΠΑΠΑΔΟΠΟΥΛΟΣ ΣΤΑΥΡΟΣ » Κυρ Φεβ 17, 2019 6:42 pm

sot arm έγραψε:
Κυρ Φεβ 17, 2019 2:55 pm
M.S.Vovos έγραψε:
Παρ Φεβ 15, 2019 6:51 pm
Έστω η συνεχής συνάρτηση f:\left [ -1,1 \right ]\longrightarrow \mathbb{R}. Να αποδείξετε ότι:

\displaystyle{\lim_{n\rightarrow \infty }\frac{\displaystyle \int_{-1}^{1}f(x)\left ( 1-x^{2} \right )^{n}\textup{d}x}{\displaystyle \int_{-1}^{1}\left ( 1-x^{2} \right )^{n}\textup{d}x}=f(0)} Φιλικά,
Μάριος



Μέχρι 17/02/2019.
Βάζω μια λύση παρακάτω με κάποια επιφύλαξη για τυχόν λογιστικό σφάλμα, έστω:

\displaystyle{L(f(x))=\lim_{n\rightarrow \infty }\frac{\displaystyle \int_{-1}^{1}f(x)\left ( 1-x^{2} \right )^{n}\textup{d}x}{\displaystyle \int_{-1}^{1}\left ( 1-x^{2} \right )^{n}\textup{d}x}}

Παρατηρώ ότι είναι γραμμική, πράγματι:

\displaystyle{L(cf+g)=cL(f)+L(g), c \in \mathbb{R}} , προκύπτει απλά με πράξεις, και:

\displaystyle{L(c)=\lim_{n\rightarrow \infty }\frac{\displaystyle \int_{-1}^{1}c\left ( 1-x^{2} \right )^{n}\textup{d}x}{\displaystyle \int_{-1}^{1}\left ( 1-x^{2} \right )^{n}\textup{d}x}=c \lim_{n\rightarrow \infty }\frac{\displaystyle \int_{-1}^{1}\left ( 1-x^{2} \right )^{n}\textup{d}x}{\displaystyle \int_{-1}^{1}\left ( 1-x^{2} \right )^{n}\textup{d}x}=c}

Εξετάζουμε τώρα μονώνυμα της μορφής x^{l} , αν l περιττός άμεσα το όριο μηδέν αφού η συνάρτηση εντός του ολοκληρώματος του αριθμητή είναι περιττή,και το ολοκλήρωμα είναι σε συμμετρικό γύρω από το 0 διάστημα.

Αν πάλι l άρτιος προκύπτει με παραγοντική:

\displaystyle{\int_{-1}^{1}x^{2k}(1-x^{2})^{n}dx=\frac{2n}{2k+1}\int_{-1}^{1}x^{2k+2}(1-x^{2})^{n}dx=...=\frac{2^{n+1}n!}{\prod_{i=0}^{n}(2k+2i+1)}}

Ο παρονομαστής είναι εφαρμογή του παραπάνω για κ=0, έχω λοιπόν για το κλάσμα:

\displaystyle{L(x^{2k})=\lim_{n\rightarrow +\infty}\frac{\prod_{i=0}^{n}(2i+1)}{\prod_{i=0}^{n}(2k+2i+1)}=\lim_{n\rightarrow +\infty}\frac{\prod_{i=1}^{n}(2i+1)}{\prod_{i=0}^{n}(2k+2i+1)}= 0}

Από τα παραπάνω για κάθε πολυώνυμο έχω:
\displaystyle{L(p(x))=p(0)}
αφού όλες οι δυνάμεις του x δίνουν μηδέν και ο σταθερός όρος ισούται με p(0)

Έστω ε>0 τυχόν, η f είναι συνεχής στο [-1,1] άρα για το δοθέν ε υπάρχει πολυώνυμο έτσι ώστε:

\displaystyle{|f(x)-p(x)|<\frac{\epsilon}{2}} (Weierstrass)

Άρα:
\displaystyle{|L(f(x)-f(0))|=|L(f(x)-f(0))-L(p(x)-p(0))|=|L(f(x)-p(x))+L(f(0)-p(0)))|\leq}

\displaystyle{|L(f(x)-p(x))|+|L(f(0)-p(0))|\leq L(|f(x)-p(x)|)+L(|f(0)-P(0)|) <\frac{\epsilon}{2}+\frac{\epsilon}{2}=\epsilon}}

και τελειώσαμε, ξαναλέω με κάποια επιφύλαξη για λογιστικό σφάλμα, αλλά νομίζω η ιδέα είναι η σωστή.
Στην σχέση
\displaystyle{\int_{-1}^{1}x^{2k}(1-x^{2})^{n}dx=\frac{2n}{2k+1}\int_{-1}^{1}x^{2k+2}(1-x^{2})^{n}dx
υπάρχει τυπογραφικό.
Είναι
\displaystyle{\int_{-1}^{1}x^{2k}(1-x^{2})^{n}dx=\frac{2n}{2k+1}\int_{-1}^{1}x^{2k+2}(1-x^{2})^{n-1}dx

το τελικό δεν νομίζω ότι έχει κάποια σημασία.



Η υπόθεση της συνέχειας της f παντού είναι περιττή.

Το αποτέλεσμα ισχύει αν η f είναι συνεχής στο 0 και έχουμε ακόμα μια συνθήκη.
π.χ f φραγμένη η
το \displaystyle \int_{-1}^{1}|f(x)|dx υπάρχει.


Άβαταρ μέλους
mikemoke
Δημοσιεύσεις: 216
Εγγραφή: Σάβ Δεκ 17, 2016 12:58 am

Re: Όριο ολοκληρωμάτων

#4

Μη αναγνωσμένη δημοσίευση από mikemoke » Τρί Φεβ 19, 2019 12:38 am

Έστω ότι f συνεχής για x=0 και φραγμένη.
s_n=\frac{\int_{-1}^{1}f(x)(1-x^2)^ndx}{\int_{-1}^{1}(1-x^2)^ndx}= \frac{\int_{-\delta }^{\delta }f(x)(1-x^2)^ndx}{\int_{-1}^{1}(1-x^2)^ndx}+\frac{\int_{A_{\delta}} f(x)(1-x^2)^ndx}{\int_{-1}^{1}(1-x^2)^ndx} (1)
όπου A_\delta =[-1,1]\setminus [-\delta ,\delta ] για \delta \in (0,1)
Ισχύεί ότι :
a)\frac{\int_{-\delta }^{\delta }(1-x^2)^ndx}{\int_{-1}^{1}(1-x^2)^n}\rightarrow 1
b)\frac{\int_{A_\delta }(1-x^2)^ndx}{\int_{-1}^{1}(1-x^2)^ndx}\rightarrow 0
Έστω \epsilon >0
τότε \exists M>0\forall x\in[-1,1]:|f(x)|\leq M
\exists \delta >0: f(0)-\epsilon /3<f(x)<f(0)+\epsilon /3
Από (1) έχουμε ότι
(f(0)-\epsilon /3)\frac{\int_{-\delta }^{\delta }(1-x^2)^ndx}{\int_{-1}^{1}(1-x^2)^ndx}+(-M)\frac{\int_{A_{\delta}} (1-x^2)^ndx}{\int_{-1}^{1}(1-x^2)^ndx} \leq s_n\leq (f(0)+\epsilon /3)\frac{\int_{-\delta }^{\delta }(1-x^2)^ndx}{\int_{-1}^{1}(1-x^2)^ndx}+M\frac{\int_{A_{\delta}} (1-x^2)^ndx}{\int_{-1}^{1}(1-x^2)^ndx}

Aπό a,b) έπεται ότι \exists n_0\in\mathbb{N}\forall n\geq n_0:f(0)-\epsilon <s_n<f(0)+\epsilon
Άρα s_n\rightarrow f(0)

Για a,b) βλέπε εδώ :https://math.stackexchange.com/question ... 18_3118095


Λάμπρος Κατσάπας
Δημοσιεύσεις: 838
Εγγραφή: Σάβ Ιουν 17, 2017 10:17 pm
Τοποθεσία: Αθήνα

Re: Όριο ολοκληρωμάτων

#5

Μη αναγνωσμένη δημοσίευση από Λάμπρος Κατσάπας » Τρί Φεβ 19, 2019 12:55 am

Την άσκηση σε όλη της τη γενικότητα μπορεί κανείς να την βρει στον Spivak, Κεφάλαιο ''Παράγωγοι και Ολοκληρώματα'' ή στον Παπαδημητράκη άσκηση 7.3. 24. Φαντάζομαι και αλλού μιας και είναι all time classic.


ΠΑΠΑΔΟΠΟΥΛΟΣ ΣΤΑΥΡΟΣ
Δημοσιεύσεις: 3600
Εγγραφή: Πέμ Φεβ 27, 2014 9:05 am
Τοποθεσία: ΧΑΛΚΙΔΑ- ΑΘΗΝΑ-ΚΡΗΤΗ

Re: Όριο ολοκληρωμάτων

#6

Μη αναγνωσμένη δημοσίευση από ΠΑΠΑΔΟΠΟΥΛΟΣ ΣΤΑΥΡΟΣ » Τρί Φεβ 19, 2019 1:50 am

Αν θέσουμε
\displaystyle p_{n}(x)=\frac{ ( 1-x^{2} )^{n}}{\int_{-1}^{1} ( 1-x^{2})^{n}dx}
θέλουμε να δείξουμε ότι
για κάθε
f:[-1,1]\rightarrow \mathbb{R}
συνεχή στο 0 και ολοκληρώσιμη
ισχύει
\displaystyle \int_{-1}^{1}f(x)p_{n}(x)dx\rightarrow f(0)

Για να το δείξουμε
αρκούν τρεις ιδιότητες που έχουν.

1)p_{n}(x)\geq 0,n=1,2,..

2)\int_{-1}^{1}p_{n}(x)dx=1,n=1,2,...

3)Για 0<\delta <1

είναι \lim_{n\rightarrow \infty }sup\left \{ |p_{n}(x)|:|x|> \delta \right \}=0

Η 1)και 2) είναι τετριμένες
Για την 3)
εχουμε
\displaystyle \int_{-1}^{1}(1-x^{2})^{n}dx\geq \int _{|x|\leq \frac{1}{\sqrt{n}}}(1-x^{2})^{n}dx\geq \frac{2}{\sqrt{n}}(1-\frac{1}{n})^{n}\geq C\frac{1}{\sqrt{n}}

οπότε

sup\left \{ |p_{n}(x)|:|x|> \delta \right \}\leq C\sqrt{n}(1-\delta ^{2})^{n}

που παίρνοντας όριο το έχουμε.

Η απόδειξη πάει ως εξης.(θα την γράψω σύντομα)

\displaystyle \int_{-1}^{1}f(x)p_{n}(x)dx- f(0)=\int_{-1}^{1}(f(x)-f(0))p_{n}(x)dx=\int _{|x|< \delta }+\int _{|x|\geq \delta }=I_{1}+I_{2}

Για το I_{1} είναι

\displaystyle |I_{1}|=|\int_{-\delta }^{\delta }(f(x)-f(0))p_{n}(x)dx|\leq \int_{-\delta }^{\delta }|f(x)-f(0)|p_{n}(x)dx
\displaystyle \leq \epsilon \int_{-\delta }^{\delta }p_{n}(x)dx\leq \epsilon \int_{-1 }^{1 }p_{n}(x)dx=\epsilon

Ενώ για το I_{2} έχουμε αν ονομάσουμε

Q_{n}=sup\left \{ |p_{n}(x)|:|x|> \delta \right \}

|I_{2}|\leq Q_{n}\int_{-1}^{1}|f(x)|dx

που προφανώς πάει στο 0

Φυσικά όλα τα παραπάνω είναι πασίγνωστα σε αυτούς που κάνουν κατανομές.
Στην ουσία η ακολουθία (p_{n})_{n\in \mathbb{N}}
συγκλίνει στην  \delta του Dirac.


Απάντηση

Επιστροφή σε “Ασκήσεις ΜΟΝΟ για φοιτητές”

Μέλη σε σύνδεση

Μέλη σε αυτήν τη Δ. Συζήτηση: Δεν υπάρχουν εγγεγραμμένα μέλη και 3 επισκέπτες